Đến nội dung

Hình ảnh

$\frac{a^3+b^3+c^3}{2abc}+\frac{a^2+b^2}{c^2+ab}+...$


  • Please log in to reply
Chủ đề này có 4 trả lời

#1
Trang Luong

Trang Luong

    Đại úy

  • Thành viên
  • 1834 Bài viết

Cho $a,b,c>0$. Chứng minh rằng : $\frac{a^3+b^3+c^3}{2abc}+\frac{a^2+b^2}{c^2+ab}+\frac{c^2+b^2}{a^2+cb}+\frac{a^2+c^2}{b^2+ac}\geq \frac{9}{2}$


Bài viết đã được chỉnh sửa nội dung bởi khonggiadinh: 14-12-2013 - 20:30

"Nếu bạn hỏi một người giỏi trượt băng làm sao để thành công, anh ta sẽ nói với bạn: ngã, đứng dậy là thành công"
Issac Newton

#2
bangbang1412

bangbang1412

    Độc cô cầu bại

  • Phó Quản lý Toán Cao cấp
  • 1667 Bài viết

Cho $a,b,c>0$. Chứng minh rằng : $\frac{a^3+b^3+c^3}{2abc}+\frac{a^2+b^2}{c^2+ab}+\frac{c^2+b^2}{a^2+cb}+\frac{a^2+c^2}{b^2+ac}\geq \frac{9}{2}$

 

$\frac{a^{2}}{2bc}+\frac{a^{2}}{c^{2}+ab}+\frac{a^{2}}{b^{2}+ac}\geq \frac{9a^{2}}{(b+c)(a+b+c)}$

Nên dùng bdt $Cauchy-schwar$ ta chỉ cần chứng minh $\sum \frac{a^{2}}{b+c}\geq \frac{\sum a}{2}$ hiển nhiên đúng 


$$[\Psi_f(\mathbb{1}_{X_{\eta}}) ] = \sum_{\varnothing \neq J} (-1)^{\left|J \right|-1} [\mathrm{M}_{X_{\sigma},c}^{\vee}(\widetilde{D}_J^{\circ} \times_k \mathbf{G}_{m,k}^{\left|J \right|-1})] \in K_0(\mathbf{SH}_{\mathfrak{M},ct}(X_{\sigma})).$$


#3
Ha Manh Huu

Ha Manh Huu

    Trung úy

  • Thành viên
  • 799 Bài viết

Cho $a,b,c>0$. Chứng minh rằng : $\frac{a^3+b^3+c^3}{2abc}+\frac{a^2+b^2}{c^2+ab}+\frac{c^2+b^2}{a^2+cb}+\frac{a^2+c^2}{b^2+ac}\geq \frac{9}{2}$

bđt phải cm tương đương với cái 3 cái sau $\geq 3$

ta có $\sum \frac{a^2+b^2}{c^2+ab}\geq \sum 2.\frac{a^2+b^2}{2c^2+a^2+b^2}$ (do $ab \leq \frac{a^2+b^2}{2}$)

đổi biến thành x,y,z ta cần cm $\sum \frac{x+y}{2z+x+y}\geq \frac{3}{2}\Leftrightarrow \sum \frac{x+y}{2z+x+y}+3\geq \frac{9}{2}$ (cộng 1 vào mỗi cái)

hay $2(x+y+z)(\sum \frac{1}{2z+x+y})\geq \frac{9}{2}$

đến đây thì dễ dang làm nốt


Bài viết đã được chỉnh sửa nội dung bởi Ha Manh Huu: 14-12-2013 - 21:12

tàn lụi


#4
Hoang Tung 126

Hoang Tung 126

    Thiếu tá

  • Thành viên
  • 2061 Bài viết

Áp dụng bđt Cosi cho 3 số có :$\sum \frac{a^2+b^2}{c^2+ab}\geq 3\sqrt[3]{\frac{(a^2+b^2)(b^2+c^2)(c^2+a^2)}{(c^2+ab)(b^2+ac)(a^2+bc)}}\geq 3$

(Do áp dụng bđt Bunhiacopxki có :$(c^2+ab)^2\leq (c^2+a^2)(c^2+b^2)$

Mà $\frac{a^3+b^3+c^3}{2abc}\geq \frac{3abc}{2abc}=\frac{3}{2}$

Đến đây cộng vế là ra



#5
Trang Luong

Trang Luong

    Đại úy

  • Thành viên
  • 1834 Bài viết

Cách khác : $\frac{a^2+b^2}{c^2+ab}=\frac{a^2c+b^2c}{c^3+abc}\geq \frac{2abc}{c^3+abc}$

$\Rightarrow \frac{a^3+b^3+c^3}{2abc}+\sum \frac{a^2+b^2}{c^2+ab}\geq \frac{a^3+b^3+c^3}{2abc}+\frac{3}{2}-\frac{3}{2}+\frac{2abc}{c^3+abc}+\frac{2abc}{b^3+abc}+\frac{2abc}{c^3+abc}\geq \frac{a^3+b^3+c^3+3abc}{2abc}+\frac{18abc}{a^3+b^3+c^3+3abc}-\frac{3}{2}\geq 6-\frac{3}{2}=\frac{9}{2}$


"Nếu bạn hỏi một người giỏi trượt băng làm sao để thành công, anh ta sẽ nói với bạn: ngã, đứng dậy là thành công"
Issac Newton




0 người đang xem chủ đề

0 thành viên, 0 khách, 0 thành viên ẩn danh